M: It is almost impossible to find a person between the ages of 85 and 90 who primarily uses the left hand.Q: Seventy...

Boram on January 9, 2019

Explanation

Can someone please explain what the question is asking? I am not understanding what it says. Thank you!

Replies
Create a free account to read and take part in forum discussions.

Already have an account? log in

zacharydtan on July 31, 2019

The question is basically asking what Q response rules out with regards to what M is trying to imply.

Irina on July 31, 2019

@Boram,

Great question. This is a tricky one.

Let's look at the argument:

M: It is almost impossible to find a person ages 85-90 who are primarily left-handed.

Now, we might be thinking why is M saying this? Is M using this fact to say that right-handed people live longer?

Q: Seventy to ninety years ago, however, children were punished for using their left hands.

Q responds by saying this fact is easy to explain because people in this age group were forced to use their right hand regardless of whether they were born right or left-handed.

The question stem asks us what is the purpose of Q's response? or what Cannot Be True given Q's response - as @zacahry aptly put it?

(A) is the correct answer choice because it points out that Q offers evidence to dispute M's hypothesis that being born right-handed confers a survival advantage.

Does this make sense?
Let me know if you have any other questions.


JackM on March 12, 2023

I still don't understand it. How are we assuming this?

Emil-Kunkin on March 18, 2023

This is indeed a tough question. It is essentially asking us what unstated idea is Q trying to argue against? While Ms statement gives us a clue, it isn't exactly clear. M tells us that there are basically no old people who are lefties. There are a number of possible causes, but the two most likely reasons for this are that lefties don't live as long (hopefully untrue!) or that something happened to lefties roughly 80 years ago that would have caused them to not be lefties. The evidence that Q provides gives us support for the latter cause, thus weakening the former cause.